Maratonas de Matemátical Maratona Olímpica de Teoria dos Números

Moderador: [ Moderadores TTB ]

Avatar do usuário
Autor do Tópico
Ittalo25
5 - Mestre
Mensagens: 2346
Registrado em: Seg 18 Nov, 2013 22:11
Última visita: 17-03-24
Out 2020 23 01:16

Re: l Maratona Olímpica de Teoria dos Números

Mensagem não lida por Ittalo25 »

Solução do problema 49

Fazendo [tex3]mdc(a,b) = d\rightarrow \begin{cases}
a=dx \\
b= dy
\end{cases}
[/tex3] , com [tex3]mdc(x,y) = 1 [/tex3] e sabendo que [tex3]mmc(a,b) \cdot mdc(a,b) = ab [/tex3]

[tex3]mmc(a,b)+mdc(a,b)=k(a+b)[/tex3]
[tex3]dxy+d=kd(x+y)[/tex3]
[tex3]\frac{4xy+4}{x+y}=4k[/tex3]

Busca-se que:
[tex3]a+b = d(x+y) \geq x+y \overbrace\geq^{?} 4k = \frac{4xy+4}{x+y} [/tex3]
[tex3]x+y \overbrace\geq^{?} \frac{4xy+4}{x+y} [/tex3]
[tex3]x^2+y^2+2xy \overbrace\geq^{?} 4xy+4 [/tex3]
[tex3](x-y)^2 \overbrace\geq^{?} 4 [/tex3]
[tex3]|x-y| \overbrace\geq^{?} 2 [/tex3]

Se isso não for verdade, então:
Caso 1:
Se [tex3]|x-y| = 0\rightarrow x = y [/tex3] , mas como [tex3]mdc(x,y) = 1 [/tex3] então [tex3]x = y = 1 [/tex3] , mas então teríamos [tex3]k=1 [/tex3] .
Caso 2:
Se [tex3]|x-y| = 1\rightarrow x = y+1 [/tex3] , teríamos:
[tex3]\frac{y^2+y+1}{2y+1}=k[/tex3]
Mas então:
[tex3]2y+1 |y^2+y+1 [/tex3]
[tex3]2y+1 |2\cdot (y^2+y+1) - y \cdot (2y+1) [/tex3]
[tex3]2y+1 |y+2 [/tex3]
[tex3]2y+1 |2\cdot (y+2) - (2y+1) [/tex3]
[tex3]2y+1 |3 [/tex3]
[tex3]y = 1\rightarrow x = 2 \rightarrow k = 1 [/tex3]

Portanto a desigualdade é verdadeira.

----------------------------------------------------------------------------------------------------------------------------

Problema 50
(Estados Unidos - 1989) Calcule [tex3]\sqrt{31 \cdot 30 \cdot 29 \cdot 28+1}[/tex3]



Ninguém pode ser perfeito, mas todos podem ser melhores. [\Bob Esponja]

Avatar do usuário
AnthonyC
4 - Sabe Tudo
Mensagens: 964
Registrado em: Sex 09 Fev, 2018 19:43
Última visita: 21-02-24
Out 2020 23 14:18

Re: l Maratona Olímpica de Teoria dos Números

Mensagem não lida por AnthonyC »

Solução do Problema 50

Lema: o produto de quatro números consecutivos somado 1 é um quadrado perfeito.
Demonstração:
Resposta

Seja [tex3]n\in \mathbb{N}[/tex3] , temos que [tex3]n-1,n,n+1[/tex3] e [tex3]n+2[/tex3] são números consecutivos. Então:
[tex3](n-1)n(n+1)(n+2)+1[/tex3]
[tex3](n^2-1)(n^2+2n)+1[/tex3]
[tex3]n^4+2n^3-n^2-2n+1[/tex3]
[tex3]n^4+[n^2-n^2]+[n^3+n^3]-n^2+[-n-n]+1[/tex3]
[tex3]n^4+n^3-n^2+n^3+n^2-n-n^2-n+1[/tex3]
[tex3]n^2(n^2+n-1)+n(n^2+n-1)-(n^2+n-1)[/tex3]
[tex3](n^2+n-1)(n^2+n-1)[/tex3]
[tex3](n^2+n-1)^2[/tex3]
C.Q.D
Utilizando o lema, temos:
[tex3]n=29[/tex3]

Portanto:
[tex3]28\cdot 29\cdot 30\cdot31+1=(29^2+29-1)^2[/tex3]
[tex3]\sqrt{28\cdot 29\cdot 30\cdot31+1}=29^2+29-1[/tex3]
[tex3]\sqrt{28\cdot 29\cdot 30\cdot31+1}=869[/tex3]
Problema 51
(Bósnia e Herzegovina- 2015)
Encontre o valor mínimo da seguinte expressão
[tex3]{a+1\over a(a+2)}+{b+1\over b(b+2)}+{c+1\over c(c+2)}[/tex3]
para [tex3]a,b,c\in \mathbb{R}_+^*[/tex3] , tais que [tex3]a+b+c\leq 3[/tex3] .

Última edição: AnthonyC (Sex 23 Out, 2020 14:19). Total de 1 vez.


[tex3]\color{YellowOrange}\textbf{Não importa o quanto se esforce ou evolua, você sempre estará abaixo do Sol}[/tex3]
[tex3]\textbf{Escanor}[/tex3]

Avatar do usuário
Autor do Tópico
Ittalo25
5 - Mestre
Mensagens: 2346
Registrado em: Seg 18 Nov, 2013 22:11
Última visita: 17-03-24
Out 2020 23 15:38

Re: l Maratona Olímpica de Teoria dos Números

Mensagem não lida por Ittalo25 »

Solução do problema 51

Abrindo as frações parciais:
[tex3]{a+1\over a(a+2)}+{b+1\over b(b+2)}+{c+1\over c(c+2)} = \frac{1}{2a}+\frac{1}{2b}+ \frac{1}{2c}+\frac{1}{2a+4}+ \frac{1}{2b+4}+\frac{1}{2c+4}[/tex3]

Usando o lema de Titu:

[tex3]\frac{1}{2a}+\frac{1}{2b}+ \frac{1}{2c}+\frac{1}{2a+4}+ \frac{1}{2b+4}+\frac{1}{2c+4} \geq \frac{(1+1+1)^2}{2(a+b+c)}+\frac{(1+1+1)^2}{2(a+b+c)+4+4+4}[/tex3]

Pela condição dada:

[tex3]\frac{(1+1+1)^2}{2(a+b+c)}+\frac{(1+1+1)^2}{2(a+b+c)+4+4+4} \geq \frac{9}{2\cdot 3 }+\frac{9}{2\cdot 3 + 12} = \boxed{2}[/tex3]

--------------------------------------------------------------------------------------------------------------------------------------

Problema 52
(Índia - 2012) Quantos pares de inteiros positivos (x,y) existem tais que [tex3]x+3y = 100 [/tex3]
Resposta

33


Ninguém pode ser perfeito, mas todos podem ser melhores. [\Bob Esponja]

Avatar do usuário
AnthonyC
4 - Sabe Tudo
Mensagens: 964
Registrado em: Sex 09 Fev, 2018 19:43
Última visita: 21-02-24
Out 2020 23 20:04

Re: l Maratona Olímpica de Teoria dos Números

Mensagem não lida por AnthonyC »

Solução do Problema 52

Sejam [tex3]x,y\in \mathbb{Z}^*_+[/tex3] . Temos:
[tex3]x+3y=100[/tex3]
[tex3]y={100-x\over3}[/tex3]
Para que tenhamos ambos inteiros, devemos ter:
[tex3]100-x\equiv0(\mod3)[/tex3]
[tex3]1-x\equiv0(\mod3)[/tex3]
[tex3]-x\equiv-1(\mod3)[/tex3]
[tex3]-x\equiv-1(\mod3)[/tex3]
[tex3]x\equiv1(\mod3)[/tex3]
[tex3]x=3k+1[/tex3]

[tex3]y={100-x\over3}[/tex3]
[tex3]y={100-3k-1\over3}[/tex3]
[tex3]y=33-k[/tex3]
Assim, para cada valor de [tex3]k[/tex3] , teremos uma solução. Sabemos que:
[tex3]x\geq1\implies k\geq0[/tex3]
[tex3]y\geq1\implies k\leq32[/tex3]
Então [tex3]0\leq k\leq32[/tex3] , o que resulta em 33 pares de solução.

Problema 53
(IMO-1981)
Determine o valor máximo de [tex3]m^2+n^2[/tex3] , onde [tex3]m[/tex3] e [tex3]n[/tex3] são inteiros que satisfazem [tex3]m,n\in \{1,2,...,1981\}[/tex3] e [tex3]\(n^2-mn-m^2\)^2=1[/tex3] .


[tex3]\color{YellowOrange}\textbf{Não importa o quanto se esforce ou evolua, você sempre estará abaixo do Sol}[/tex3]
[tex3]\textbf{Escanor}[/tex3]

Avatar do usuário
Autor do Tópico
Ittalo25
5 - Mestre
Mensagens: 2346
Registrado em: Seg 18 Nov, 2013 22:11
Última visita: 17-03-24
Out 2020 25 01:01

Re: l Maratona Olímpica de Teoria dos Números

Mensagem não lida por Ittalo25 »

Solução do problema 53

Para [tex3]m = n\rightarrow (n^2-n^2+n^2)^2 = 1[/tex3] , então [tex3]n=1 [/tex3] e [tex3]m=1 [/tex3] , [tex3]n^2+m^2=2[/tex3]
Fora isso, é fácil ver que [tex3]n>m [/tex3] , do contrário:
[tex3]n^2-mn-m^2 = n\cdot (n-m) - m^2 [/tex3] seria a soma de 2 números negativos e portanto daria resultado menor que -1

Então [tex3]n>m [/tex3] , fazendo [tex3]n = m+x [/tex3] para algum inteiro positivo x
[tex3]\(n^2-mn-m^2\)^2=1[/tex3]
[tex3]\((m+x)^2-m(m+x)-m^2\)^2=(-(m^2-mx-x^2))^2 = (m^2-mx-x^2)^2 = 1[/tex3]
Portanto uma solução [tex3](n,m) [/tex3] dá uma solução menor [tex3](m,x) = (m,n-m) [/tex3] . Esse processo tem que chegar até a menor solução possível.

A menor solução é [tex3](n,m) = (2,1) [/tex3]
E então basta usar: [tex3](n,m) \rightarrow (m,n-m) [/tex3]
[tex3](2,1)\rightarrow (2,3)\rightarrow (3,5)\rightarrow (5,8)\rightarrow .....\rightarrow (987,1597) [/tex3]

Sendo assim, o valor máximo de [tex3]\boxed{ m^2+n^2 = 987^2+1597^2} [/tex3]

-----------------------------------------------------------------------------------------------------------------------------------------

Problema 54
(Estados Unidos - 2013) O número 2013 tem a propriedade de que o seu dígito das unidades é igual a soma dos seus outros dígitos, ou seja: [tex3]3 = 2+0+1 [/tex3] . Quantos inteiros menores que 2013 e maiores que 1000 tem essa propriedade?
Resposta

46


Ninguém pode ser perfeito, mas todos podem ser melhores. [\Bob Esponja]

Avatar do usuário
AnthonyC
4 - Sabe Tudo
Mensagens: 964
Registrado em: Sex 09 Fev, 2018 19:43
Última visita: 21-02-24
Out 2020 26 19:04

Re: l Maratona Olímpica de Teoria dos Números

Mensagem não lida por AnthonyC »

Solução do Problema 54
Utilizarei a notação de barra para separação de dígitos. Ex: [tex3]123=1\|2\|3[/tex3]
Seja [tex3]n\in \mathbb{N}[/tex3] , tal que [tex3]1001\leq n\le 2012[/tex3] . Se ele possuí a propriedade de 2013, então podemos escrevê-lo como:
[tex3]n=a\|b\|c\|(a+b+c)[/tex3]
onde [tex3]a,b,c\in\mathbb{N}[/tex3] e [tex3]0\leq a,b,c\leq9[/tex3] , com [tex3]a\neq0[/tex3] .
Dado o intervalo de valores para [tex3]n[/tex3] , temos que [tex3]a=1\text{ ou } 2[/tex3] .
  • [tex3]a=1[/tex3]
[tex3]n=1\|b\|c\|(1+b+c)[/tex3]
Como o algarismos das unidades está entre 0 e 9:
[tex3]0\leq 1+b+c\le 9[/tex3]
[tex3]-1\leq b+c\le 8[/tex3]
Porém, [tex3]b,c\geq0[/tex3] , então:
[tex3]0\leq b+c\le 8[/tex3]
Como ambos são não-negativos, devemos ter [tex3]b,c\leq8[/tex3] . Por fim, temos que:
[tex3]-c\leq b\le 8-c[/tex3]
Novamente podemos afirmar que:
[tex3]0\leq b\le 8-c[/tex3]
Assim, para cada valor de [tex3]c[/tex3] , termos um certo intervalo de valores para [tex3]b[/tex3] , mas precisamente, terremos [tex3]8-c+1[/tex3] , valores. Assim, como [tex3]0\leq c\leq 8[/tex3] , o número de valores para [tex3]b[/tex3] será [tex3]\text{nº}(b)=\{9,8,7,...,2,1,0\}[/tex3] . Somando todos estes, temos que o existem 45 números com a propriedade desejada para [tex3]a=1[/tex3] ;
  • [tex3]a=2[/tex3]
  • [tex3]2000\leq n\lt2010[/tex3] , temos [tex3]b=c=0[/tex3] , o que implica [tex3]n=2\|0\|0\|2=2002[/tex3]
  • [tex3]n\in \{2010,2011,2012\}[/tex3] , temos [tex3]b=0[/tex3] e [tex3]a+c=3[/tex3] , porém a unidade é menor que 3, então não temos solução aqui.
Assim, os 45 anteriores, somados com o 2002, resultam em 46 números com a propriedade desejada;

Problema 55
(IMO-1998)
Determine todos os pares de inteiros positivos [tex3](a,b)[/tex3] tais que [tex3]ab^2+b+7[/tex3] divide [tex3]a^2b+a+b[/tex3] .


[tex3]\color{YellowOrange}\textbf{Não importa o quanto se esforce ou evolua, você sempre estará abaixo do Sol}[/tex3]
[tex3]\textbf{Escanor}[/tex3]

Avatar do usuário
Autor do Tópico
Ittalo25
5 - Mestre
Mensagens: 2346
Registrado em: Seg 18 Nov, 2013 22:11
Última visita: 17-03-24
Out 2020 30 00:40

Re: l Maratona Olímpica de Teoria dos Números

Mensagem não lida por Ittalo25 »

Solução do problema 55

[tex3]ab^2+b+7|a^2b+a+b [/tex3]
[tex3]ab^2+b+7|b\cdot (a^2b+a+b) - a\cdot (ab^2+b+7) [/tex3]
[tex3]ab^2+b+7|b^2-7a [/tex3]

Se [tex3]b^2-7a = 0 [/tex3] , então [tex3](a,b) = (7x^2,7x) [/tex3] é sempre solução.

Do contrário: [tex3]ab^2+b+7\leq |b^2-7a| [/tex3]

Se [tex3]b^2-7a > 0 [/tex3] , não há solução porque: [tex3]ab^2+b+7 > b^2 > b^2-7a [/tex3]

Então [tex3]b^2-7a < 0 [/tex3] :
[tex3]ab^2+b+7\leq 7a-b^2 [/tex3]
[tex3]ab^2<ab^2+b+7+b^2 \leq 7a [/tex3]
[tex3]ab^2< 7a [/tex3]
[tex3]b \leq 2 [/tex3]

Para [tex3]b=1[/tex3]
[tex3]a+8|a^2+a+1 [/tex3]
[tex3]a+8|a^2+a+1 -a\cdot (a+8)[/tex3]
[tex3]a+8|7a-1[/tex3]
[tex3]a+8|7a-1 - 7 \cdot (a+8)[/tex3]
[tex3]a+8|57[/tex3]

Para [tex3]b=2[/tex3]
[tex3]4a+9| 4-7a [/tex3]
[tex3]4a+9| 4\cdot (4-7a) + 7 \cdot (4a+9) [/tex3]
[tex3]4a+9|79 [/tex3]

As soluções ficam: [tex3]\boxed{(a,b) = (7x^2,7x), (1,49),(1,11)}[/tex3]

----------------------------------------------------------------------------------------------------------------------------------------------

Problema 56
(Índia 2019) Seja [tex3]\overline{abc}[/tex3] um número de 3 dígitos onde [tex3]a,b,c\neq 0[/tex3] e tal que [tex3]a^2+b^2=c^2 [/tex3] . Qual o maior primo possível que divide [tex3]\overline{abc}[/tex3] ?
Resposta

29


Ninguém pode ser perfeito, mas todos podem ser melhores. [\Bob Esponja]

Avatar do usuário
NigrumCibum
2 - Nerd
Mensagens: 356
Registrado em: Sáb 31 Out, 2020 16:02
Última visita: 16-11-23
Nov 2020 24 17:34

Re: l Maratona Olímpica de Teoria dos Números

Mensagem não lida por NigrumCibum »

Solução do problema 56

Teorema: Todas as soluções primitivas de [tex3]a^2+b^2=c^2[/tex3] com b par, são da forma [tex3]a=r^2-s^2[/tex3] , [tex3]b=2rs[/tex3] e [tex3]c=r^2+s^2[/tex3] , onde r e s são inteiros de paridade oposta com [tex3]r>s>0[/tex3] e [tex3]mdc(r, ~s)=1.[/tex3]
Admitindo [tex3]r=2[/tex3] e [tex3]s=1[/tex3] , tem-se que:
[tex3]a=3 ~ou~(4), ~b=4~ou~(3)~~e~~c=5[/tex3]
É fácil perceber que essas são as únicas soluções onde a, b e c menores que 10 e maiores que 0, portanto: [tex3]\overline {abc}∈(345, ~435).[/tex3] Os divisores primos de 345 são 3, 5 e 23, os divisores primos de 435 são 3, 5 e 29. Portanto, o maior divisor primo de [tex3]\overline{abc}[/tex3] é 29.

Problema 57

(IMO-1968) Para todo número natural n, calcule o valor da soma: [tex3]⌊{n+1\over 2}⌋+⌊{n+2\over 4}⌋+⌊{n+4\over 8}⌋+⌊{n+8\over 16}⌋+...+⌊{n+2^k\over 2^{k+1}}⌋+...[/tex3]
Resposta

n
Última edição: NigrumCibum (Ter 24 Nov, 2020 17:52). Total de 3 vezes.


Arrêter le temps!

Avatar do usuário
Autor do Tópico
Ittalo25
5 - Mestre
Mensagens: 2346
Registrado em: Seg 18 Nov, 2013 22:11
Última visita: 17-03-24
Nov 2020 30 23:39

Re: l Maratona Olímpica de Teoria dos Números

Mensagem não lida por Ittalo25 »

Problema 57 removido para IMO - 1968 - Função piso
Conforme a regra número 6.

__________________________________________________________________________________________________________

Problema 58

(Áustria - 2016) Encontre todos inteiros positivos k e n que satisfazem [tex3]k^2-2016=3^n [/tex3]
Resposta

(n,k)=(2,45)
Última edição: Ittalo25 (Seg 28 Dez, 2020 01:29). Total de 3 vezes.


Ninguém pode ser perfeito, mas todos podem ser melhores. [\Bob Esponja]

Deleted User 25040
6 - Doutor
Última visita: 31-12-69
Dez 2020 29 12:15

Re: l Maratona Olímpica de Teoria dos Números

Mensagem não lida por Deleted User 25040 »

Solução do Problema 58
primeiro eu olhei para os quadrados perfeitos mais próximos de 2016, [tex3]45^2=2025[/tex3]
se subtrairmos 9 de ambos os lados vamos ter o seguinte
[tex3]k^2-45^2=3^n-3^2[/tex3]
[tex3](k-45)(k+45)=3^n-3^2~~~(i)[/tex3]
o lado direito vai ser divisível por 3 logo o lado esquerdo tbm sera.
[tex3]k=3z+r[/tex3] com [tex3]0\leq r < 3[/tex3] daí
[tex3]3|3z+r-45 [/tex3] ou [tex3]3|3z+r+45[/tex3] já que 3 é primo, em ambos os casos concluímos que [tex3]3|r[/tex3] logo r = 0
suponha por absrudo que exista solução para n > 2
[tex3]9z^2-2016=3^n[/tex3]
[tex3]z^2-2^5\cdot7=3^{n-2}[/tex3] como estamos considerando n > 2, o lado direito é divisível por 3 então
[tex3]z^2-2^5\cdot7\equiv0(\mod3)[/tex3]
[tex3]z^2\equiv2(\mod3)[/tex3] mas 2 não é resíduo quadrático módulo 3
logo não há solução para n maior que 2
1º caso: n = 2
[tex3]k^2-2016=9[/tex3]
[tex3]k^2=45^2[/tex3] como queremos soluções inteiras e positivas k = 45 e n = 2 é uma solução
2º caso: n = 1
[tex3]k^2-2016 = 3[/tex3]
[tex3]k^2=2019[/tex3] e 2019 não é um quadrado perfeito, logo a única solução é k = 45 e n = 2
problema 59
(Brasil - 2011) Quantos são os pares ordenados (a,b), com a, b inteiros positivos, tais que
[tex3]a + b + mdc(a,b) = 33[/tex3] ?




Responder
  • Tópicos Semelhantes
    Respostas
    Exibições
    Última msg

Voltar para “Maratonas de Matemática”